16 votos

Estados límite del $V(x)=\pm \delta'^{(n)}(x)$ ¿potencial?

En $\delta(x)$ La delta de Dirac no es el único potencial "puntual" que podemos integrar; en principio todas sus derivadas $\delta', \delta'', ...$ también existen, ¿verdad?

En caso afirmativo, ¿podemos buscar estados vinculados en alguno de estos $\delta'^{(n)}(x)$ ¿potenciales? ¿Existen fórmulas explícitas para ellos (y para los estados de dispersión)?

Para ser más preciso, pido soluciones explícitas de la ecuación de Schroedinger 1D con potencial puntual,

$$- {\hbar^2 \over 2m} \Psi_n''(x) + a \ \delta'^{(n)}(x) \Psi(x) \ = E_n \Psi_n(x) $$

Debo añadir que he leído al menos de tres conjunto de condiciones de contorno que se dice que son soluciones particulares:

  • $\Psi'(0^+)-\Psi'(0^-)= A \Psi(0)$ con $\Psi(0)$ continua, es la derivada cero, el "potencial delta".
  • $\Psi(0^+)-\Psi(0^-)= B \Psi'(0)$ con $\Psi'(0)$ continua, fue llamado "el potencial delta primo" por Holden.
  • $\lambda \Psi'(0^+)=\Psi'(0^-)$ y $\Psi(0^+)=\lambda\Psi(0^-)$ simultáneamente, fue llamado "el potencial delta primo" por Kurasov

El caso de la derivada cero, $V(x)=a \delta(x)$ es un ejemplo típico de libro de texto, bastante bonito porque sólo tiene un estado límite, por negativo $a$ y actúa como una especie de barrera para los $a$ . Por eso es interesante preguntarse por otros valores de $n$ y, por supuesto, para el caso general y si ofrece más estados vinculados u otras propiedades. ¿Es posible considerar $n$ más allá de la primera derivada?

Preguntas relacionadas

( Si recuerdas alguna pregunta relacionada, no dudes en sugerirla en los comentarios, o incluso editarla directamente si tienes privilegios para ello )

Para la prima delta, incluidos los potenciales dependientes de la velocidad, se ha planteado la pregunta en ¿Cómo interpretar la derivada del potencial delta de Dirac?

En la línea media $r>0$ El delta se denomina "pseudopotencial de Fermi". A día de hoy no puedo ver preguntas al respecto, pero Límite clásico de un sistema cuántico parece tener el mismo potencial.

Una forma general de tomar con boundaring condiciones es a través de la teoría de autoadjunto extensiones de operadores hermitianos. Este caso no es muy diferente de la "partícula en caja 1D", cuestión ¿Por qué $ \psi = A \cos(kx) $ no es una función de onda aceptable para una partícula en una caja? Una toma general fue la pregunta Interpretación física de distintas extensiones autoadjuntas Una pregunta relacionada pero muy exótica es ¿Cuál es la relación entre la renormalización y la extensión autoadjunta? porque, obviamente, las interacciones apoyadas en puntos tienen un escalado peculiar

Comentarios

Por supuesto actualizar las distribuciones para buscar como operadores en $L^2$ es delicado, y empeora para las derivadas de distribuciones cuando se considera su evaluación $<\phi | \rho(x) \psi>$ . Consideremos el caso $\rho(x) = \delta'(x) = \delta(x) {d\over dx}$ . ¿Debe aplicarse el derivado a $\psi$ únicamente, o al producto $\phi^*\psi$ ?

1 votos

Es difícil crear condiciones límite en la barrera. Integrando ambos lados de la ecuación infinitesimalmente se obtiene $-\frac{\hbar^2}{2m}[\Psi^{'}_{+}(0)-\Psi^{'}_{-}(0)]=a\Psi^{'}(0)$ pero no estoy seguro de cómo interpretarlo.

0 votos

@JahanClaes No estoy seguro de la $a=0$ caso, debido a la $0 \ \infty$ indeterminación, pero sí para $\delta'^{(0)}$ y $a \neq 0$ este es el argumento habitual en los libros de texto, que la ecuación equivale a exigir condiciones de contorno $\Psi'(0^+)-\Psi'(0^-) \propto \Psi(0)$ . Para $ n > 0$ , si la primera derivada es continua entonces su fórmula conduce a una condición $\Psi'^{(n)}=0$ , y estoy de acuerdo en que no está claro cómo interpretarlo, y si es la solución más general.

2 votos

Sí, pero por supuesto no estoy seguro de que podamos exigir que $\Psi^{'}$ es continua en 0, ya que no lo es para a $\delta$ -potencial de función.

7voto

Jahan Claes Puntos 1026

Ok, tengo una solución para $\delta'(x)$ basado en un límite muy burdo. Voy a descuidar los factores de $\hbar$ , $m$ etc. para eliminar el desorden.

Sea $V_\epsilon(x)=\frac{\delta(x+\epsilon)-\delta(x)}{\epsilon}$ . Entonces $\lim_{\epsilon\rightarrow 0}V_e(x)=\delta'(x)$ . Resolveremos la ecuación de Schrodinger para finito $\epsilon$ y luego tomar el límite después.

Tenemos $\Psi''(x) = (E-V_\epsilon(x))\Psi(x)$ . Si queremos una solución acotada, debemos tener $E<0$ . A continuación, en los rangos $[-\infty,0),(0,\epsilon),(\epsilon,\infty]$ debemos tener eso $\Psi$ es alguna función exponencial. En otras palabras, $$ \Psi(x) =\left\{\begin{array}{ll} e^{\sqrt{-E}x} &x\in [-\infty,0)\\ Ae^{\sqrt{-E}x}+Be^{-\sqrt{-E}x} &x\in (0,\epsilon)\\ Ce^{-\sqrt{-E}x}&x\in (\epsilon,\infty]\\ \end{array}\right\} $$

Del hecho de que $\Psi$ debe ser continua, podemos sustituir $B$ y $C$ en términos de $A$ para obtener $$ \Psi(x) =\left\{\begin{array}{ll} e^{\sqrt{-E}x} &x\in [-\infty,0)\\ Ae^{\sqrt{-E}x}+(1-A)e^{-\sqrt{-E}x} &x\in (0,\epsilon)\\ (1-A+Ae^{2\sqrt{-E}\epsilon})e^{-\sqrt{-E}x}&x\in (\epsilon,\infty]\\ \end{array}\right\} $$

También podemos escribir la derivada de $\Psi$ .

$$ \Psi'(x) =\left\{\begin{array}{ll} \sqrt{-E}e^{\sqrt{-E}x} &x\in [-\infty,0)\\ A\sqrt{-E}e^{\sqrt{-E}x}+(A-1)\sqrt{-E}e^{-\sqrt{-E}x} &x\in (0,\epsilon)\\ (A-1-Ae^{2\sqrt{-E}\epsilon})\sqrt{-E}e^{-\sqrt{-E}x}&x\in (\epsilon,\infty]\\ \end{array}\right\} $$

Utilizando el método normal para encontrar las condiciones de contorno en un $\delta$ tenemos que

$$ \begin{array}{c} \Psi'_{+}(0)-\Psi'_{-}(0) = \Psi(0) \\ \Psi'_{+}(\epsilon)-\Psi'_{-}(\epsilon) = \Psi(\epsilon) \end{array} $$

La primera condición de contorno nos da

$$ (2A-1)\sqrt{-E} = \frac{1}{\epsilon}$$ o $$ A=\frac{1}{2\epsilon\sqrt{-E}}+\frac{1}{2} $$

La segunda condición de contorno nos da

$$ -2A\sqrt{-E}e^{\sqrt{-E}\epsilon} = -\frac{1}{\epsilon}[Ae^{\sqrt{-E}\epsilon}+(1-A)e^{-\sqrt{-E}\epsilon}] $$ o $$ A=\frac{1}{e^{2\sqrt{-E}\epsilon}(2\epsilon\sqrt{-E}-1)+1} $$

Juntando las dos condiciones obtenemos una restricción sobre $E$ .

$$ \frac{1}{2\epsilon\sqrt{-E}}+\frac{1}{2} = \frac{1}{e^{2\sqrt{-E}\epsilon}(2\epsilon\sqrt{-E}-1)+1} $$

Podemos expandir ambos lados en una serie de Laurent de primer orden en $\epsilon$ . El lado izquierdo ya está expandido. El lado derecho se convierte (en primer orden) en

$$ \frac{1}{(2\epsilon\sqrt{-E}+1)(2\epsilon\sqrt{-E}-1)+1}=\frac{1}{4\epsilon^2(-E)} $$

Los dos lados de la ecuación son entonces imposibles de igualar en el límite $\epsilon\rightarrow 0$ ya que se producen en órdenes diferentes en $\frac{1}{\epsilon}$ . Por lo tanto, en ese límite, ninguna solución para $E$ existe, por lo que no hay estado límite.

Seguro que hay algún error de álgebra en todo ese lío, pero esa es la idea general. Podrías hacer el mismo álgebra para ver los estados de dispersión, si quisieras. También se podría aplicar este método a derivadas superiores. Por ejemplo, $\delta''(0)=\lim_{\epsilon\rightarrow 0}\frac{\delta(x+2\epsilon)-2\delta(x+\epsilon)+\delta(x)}{\epsilon^2}$ . Por supuesto, cada derivada más alta exige un límite más a tener en cuenta, por lo que el problema se complica. Pero, en principio, es factible.

0 votos

Una especie de horquilla de Dirac en lugar de peine de Dirac. Me gusta el planteamiento, sobre todo para derivadas más altas del delta... es bueno que sean más condiciones de contorno porque a medida que uno aumenta $n$ uno de los enigmas era cómo quedarse sólo con las condiciones de segundo orden.

1 votos

Me he puesto a revisar la bibliografía y se ha intentado resolver el $\epsilon \to 0$ conflicto utilizando el "Límite Resolvente Fuerte" de la secuencia de hamiltonianos. Sea lo que sea, el resultado no fue alentador, equivale a condiciones de Dirichlet.

0 votos

+1, pero en realidad se pueden tomar las derivadas delta "normales" para darse cuenta de que no hay forma de igualar derivadas superiores de dos exponenciales que caen a ambos lados (también conocidas como estados límite). Esto también tiene sentido físico cuando se piensa en las deltas en términos de expansiones multipolares; la ausencia de la mera $\delta$ significa que la fuente no tiene "monopolo global", es decir, que no tiene "potencial de enlace global".

6voto

John R Ramsden Puntos 143

Simplifiquemos las cosas, $\hbar=m=1$ y ponemos el $\delta^{(n)}$ como $V(x)=V_0\delta^{(n)}/2$ . Entonces el problema es $$-\psi'' - V_0\delta^{(n)}\psi = E \psi$$ Además de $x=0$ la ecuación da $$\psi'' = -E \psi $$ queremos un estado ligado $E<0$ que cae en el infinito, por lo que obtenemos una solución $\psi_+ = A \exp(-k x)$ para $x>0$ y $\psi_- = A \exp(kx)$ para $x<0$ con $k = \sqrt{-E}$ . Ahora tenemos que coser la solución alrededor de $x=0$ . La única solución no trivial declarada tiene $\psi \sim$ cúspide, $\psi' \sim$ salta, $\psi'' \sim -\delta$ , $\psi''' \sim -\delta'$ ... alrededor $x=0$ . Al introducir los valores no triviales $\psi_-,\psi_+$ en nuestra ecuación dinámica encontramos $$2k^2 A \delta - A V_0 \delta^{(n)} = 0$$ que nos da

  1. No hay solución para $n \neq 0$
  2. Un único estado límite $E=-V_0/2$ para $n=0$ (¡véase que el 1/2 en la definición del potencial contraataca!); $A$ viene determinada por la normalización de la función de onda.

Ahora ves que realmente no hay solución para $\delta$ -potenciales derivados, al menos en una dimensión. Como ya se ha comentado de alguna manera en los comentarios, esto también se puede ver en el hecho de que todos los $\delta$ -derivados parecen "multipicos" de algún tipo, sin ninguna unión "global".

Para entender mejor lo que quiero decir, considere $\delta'$ que puede obtenerse mediante un proceso límite de la derivada de una gaussiana: enter image description here

Por tanto, es físicamente intuitivo que, aunque haya estados ligados en el pozo de la derecha, se eliminen de algún modo por la compresión infinita de este doble pico.

0 votos

Hmm physics.stackexchange.com/questions/143630/ dado que si $\int V(x) < 0$ hemos concedido la existencia de estados límite, tengo la impresión de que estamos con algún tipo de caso límite... ¿quizás con un estado E=0?

1 votos

No estoy diciendo que cada función del proceso límite no tenga estados límite, los tiene, y eso sería fácil de demostrar. Es sólo que el pleno $\delta$ -derivado no lo hace, porque el $\psi_+,\psi_-$ estado puede hacer esencialmente sólo una cúspide correspondiente a un $\delta$ potencial. (Obsérvese también que, a diferencia del teorema que citas, la limitación $V(x)$ es no no negativo, $\int V(x) = 0$ que no es $<0$ y aquí se trata de distribuciones, por lo que afirmaciones como $V(x)><0$ no tienen un buen significado).

1voto

kamens Puntos 6043

El artículo de D J Griffiths supone que la interacción delta puede aproximarse mediante una secuencia de funciones pares y, a continuación, infiere dos condiciones de contorno: $$ \Psi'(0^+)-\Psi'(0^-)= (-1)^n {m c \over \hbar^2} (\Psi'^{(n)}(0^+)+\Psi'^{(n)}(0^-))$$ $$ \Psi(0^+)-\Psi(0^-)= (-1)^{n-1} {m c \over \hbar^2} n (\Psi'^{(n-1)}(0^+)+\Psi'^{(n-1)}(0^-))$$

Mi opinión viaja por otros derroteros. Integrando $$- {\hbar^2 \over 2m} \Psi''(x) + a \ V(x) \Psi(x) \ = \lambda \Psi (x) $$ de $-\epsilon < 0$ a $u$ obtenemos

$$- {\hbar^2 \over 2m} (\Psi'(u) -\Psi'(-\epsilon)) + a \int_{-\epsilon}^u \ V(x) \Psi(x) \ = \int_{-\epsilon}^u \lambda \Psi (x) $$

Integrar de nuevo $u$ de $-\epsilon < 0$ a $v$ $$- {\hbar^2 \over 2m} (\Psi(v)-\Psi(-\epsilon) - (v+\epsilon) \Psi'(-\epsilon)) + a \int_{-\epsilon}^v du \int_{-\epsilon}^u dx \ V(x) \Psi(x) \ = \int_{-\epsilon}^v \int_{-\epsilon}^u \lambda \Psi (x) $$

la primera integración da la condición de contorno en el límite $$- {\hbar^2 \over 2m} (\Psi'(0^+) -\Psi'(0^-)) + a <\rho |\Psi(x)> \ = 0 $$

Para la segunda ecuación, en lugar de pasar por múltiples integraciones por partes para cada derivada, creo que podemos usarlo una sola vez:

$$ {d\over du} ( u \int_{-\epsilon}^u dx \ V(x) \Psi(x)) = \int_{-\epsilon}^u dx \ V(x) \Psi(x) + u V(u) \Psi(u) $$

$$ ( u \int_{-\epsilon}^u dx \ V(x) \Psi(x))|^v_{-\epsilon} = \int_{-\epsilon}^v\int_{-\epsilon}^u dx \ V(x) \Psi(x) +\int_{-\epsilon}^v u V(u) \Psi(u) $$

De modo que el límite de la segunda integración produce la condición de contorno

$$- {\hbar^2 \over 2m} (\Psi(0^+) -\Psi(0^-)) - a <\rho |x \Psi(x)> \ = 0 $$

Es inmediatamente visible que para $\rho = \delta'^{(n)}(x)$ mi resultado difiere que difiere de Griffiths en la alternancia de signos $(-1)^n$ . Esta es la única discrepancia (el $n$ en la segunda condición aparece de forma natural, dado que $< \delta'^{(n)}(x) | x f(x)>$ es sólo el delta sobre $x f'^{(n)} + n f'^{(n-1)}$ ) por lo que podría tratarse simplemente de algún problema en la definición de la derivada n-ésima del delta.

En cualquier caso, son objeciones más evidentes contra el resultado: requiere estar pendiente de la regularización para asegurarse de que todas las distribuciones se aplican a n-ésimas derivadas promediando a izquierda y derecha; no cubre todas las posibles condiciones de contorno de una interacción puntual -bueno, no se esperaba que las cubriera todas-, y lo peor de todo, para mí, introduce condiciones de contorno con derivadas mayores que la primera, cuando simplemente estamos resolviendo una ecuación diferencial de segundo orden. Pregunta: ¿son compatibles estas condiciones de contorno con un hamiltoniano autoadjunto? Yo pensaría que no.

Veamos ahora la matriz de dispersión. Aplicamos las condiciones de contorno a una función $\psi_k(x<0)=e^{ikx}+R e^{-ikx}, \psi_k(x>0)=T e^{ikx}$ de modo que $$\psi'^{(n)}_k(0^-)=(ik)^n (1+(-1)^n R), \psi_k^{(n)}(0^+)=T (ik) ^n$$

El BC, por $n \geq 1$ Resuélvelo:

$$- {\hbar^2 \over 2m} ik (T-1+R) + \frac a2 (ik)^n (T+1+(-1)^n R) \ = 0 $$ $$- {\hbar^2 \over 2m} (T-1-R) + \frac a2 n (ik)^{n-1} (-T-1+(-1)^n R) \ = 0 $$

mientras que para el delta habitual, $n=0$ ,

$$- {\hbar^2 \over 2m} ik (T-1+R) + \frac a2 (T+1+ R) \ = 0 $$ $$- {\hbar^2 \over 2m} (T-1-R) \ = 0 $$

En este caso, si resolvemos el coeficiente de transmisión:

$$ T = {ik \over ik- {a m /\hbar^2 } } $$

y ver que tiene un polo, en $ik= \frac a2 {2m \over \hbar^2}$ correspondiente al estado límite. No todos los polos son estados límite, pero todos los estados límite son polos, por lo que esta técnica puede ser útil para extraer información también en el $n>1$ caso.

Ah, nótese que la conservación de la corriente de probabilidad implica el requisito extra $T^2+R^2=1$ . Esto podría utilizarse para comprobar si la solución es coherente. Es fácil comprobar que esta condición no funcionará para $n>1$ ya que en este caso podemos pivotar sobre $ik$ en ambas ecuaciones para producir la restricción adicional

$$n (T-1+R)(-T-1+(-1)^n R) = (T-1-R)(T+1+(-1)^n R) $$

que se reduce a: $$n(-T^2+(-1)^n TR+1-(-1)^n R-TR-R+(-1)^n R^2) = (+T^2+(-1)^n TR-1-(-1)^n R-RT-R-(-1)^n R^2) $$

para $n$ incluso

$$n(-T^2+1-2R+ R^2) = (+T^2-1-2R- R^2) $$

para $n$ impar $$n(-T^2- 2 TR+1- R^2) = (+T^2- 2 TR-1+ R^2) $$

Para ver la incompatibilidad con la preservación de la probabilidad imponemos simultáneamente esta condición, suma de $R^2$ y $T^2$ igual a 1 y obtenemos

  • para $n$ incluso: $n(-1+ R)R = (-1- R)R $ y luego $R$ y $T$ debe ser independiente de $k$ lo que no puede ser si $n>1$
  • para $n$ impar: $nTR = TR $ y $RT=0$ o $n=1$ pero $R$ o $T$ igual a cero fija el otro a 1, y entonces de nuevo son independientes de $k$ lo que no puede ser si $n>1$

En conclusión, la aproximación de Griffiths permite dar sentido a todas las derivadas de la delta, pero las condiciones de contorno resultantes fugan probabilidad para $n>1$

0 votos

Sin embargo, las fugas de probabilidad, o la evolución no unitaria, podrían ser útiles para describir situaciones inestables. Así que los deltas producen una interesante familia de sumideros de un punto

i-Ciencias.com

I-Ciencias es una comunidad de estudiantes y amantes de la ciencia en la que puedes resolver tus problemas y dudas.
Puedes consultar las preguntas de otros usuarios, hacer tus propias preguntas o resolver las de los demás.

Powered by:

X